LSAT and Law School Admissions Forum

Get expert LSAT preparation and law school admissions advice from PowerScore Test Preparation.

User avatar
 Dave Killoran
PowerScore Staff
  • PowerScore Staff
  • Posts: 5852
  • Joined: Mar 25, 2011
|
#72704
This game is also discussed in our Podcast, at the 14:37 mark: LSAT Podcast Episode 37: The November 2019 LSAT Logic Games Section

Complete Question Explanation
(The complete setup for this game can be found here: https://forum.powerscore.com/lsat/viewtopic.php?t=31706)

The correct answer choice is (A)

Answer choice (A): This is the correct answer choice. This answer is possible only when month 3 is empty (X), but otherwise it would meet all the conditions of the game.

Answer choice (B): This answer is incorrect because it violates the first rule of the game because U is month 6 whereas month 6 must be empty.

Answer choice (C): This answer is incorrect because it violates the third rule of the game because Q and T are in months 1 and 3 instead of being consecutive.

Answer choice (D): This answer is incorrect because it violates the last rule of the game. Since all five agents are assigned months, we can infer that months 6 and 2 are empty. But, when month 2 is empty, then S must be trained in month 4, which does not occur in this answer.

Answer choice (E): This answer is incorrect because it violates the second rule of the game, and R is trained in an earlier month than U.

Note how, as often occurs in List questions, each of the wrong answer choices violates a different rule of the game.

Get the most out of your LSAT Prep Plus subscription.

Analyze and track your performance with our Testing and Analytics Package.